Quantcast
  • Register
PhysicsOverflow is a next-generation academic platform for physicists and astronomers, including a community peer review system and a postgraduate-level discussion forum analogous to MathOverflow.

Welcome to PhysicsOverflow! PhysicsOverflow is an open platform for community peer review and graduate-level Physics discussion.

Please help promote PhysicsOverflow ads elsewhere if you like it.

News

PO is now at the Physics Department of Bielefeld University!

New printer friendly PO pages!

Migration to Bielefeld University was successful!

Please vote for this year's PhysicsOverflow ads!

Please do help out in categorising submissions. Submit a paper to PhysicsOverflow!

... see more

Tools for paper authors

Submit paper
Claim Paper Authorship

Tools for SE users

Search User
Reclaim SE Account
Request Account Merger
Nativise imported posts
Claim post (deleted users)
Import SE post

Users whose questions have been imported from Physics Stack Exchange, Theoretical Physics Stack Exchange, or any other Stack Exchange site are kindly requested to reclaim their account and not to register as a new user.

Public \(\beta\) tools

Report a bug with a feature
Request a new functionality
404 page design
Send feedback

Attributions

(propose a free ad)

Site Statistics

205 submissions , 163 unreviewed
5,047 questions , 2,200 unanswered
5,345 answers , 22,709 comments
1,470 users with positive rep
816 active unimported users
More ...

  Do two charges moving in the same direction violate Einstein's Theory of Relativity?

+ 0 like - 0 dislike
2260 views

We have two massive objects in space, each with mass $m$ and charge $q$, of the same sign, and let the mass to charge ratio be $\sqrt{\frac{1}{4\pi \epsilon _0 G}}$, so that the attractive gravitational force cancel out the repulsive electrostatic force $\bigg( \frac{Gm^2}{r^2}=\frac{q^2}{4\pi \epsilon_0 r^2} \bigg)$, and there will be no net force on the objects.

Now assume we have the same setup, but the objects are moving at a speed v relative to your reference frame. Then in addition to the gravitational and electrostatic forces, we also have the magnetic force, which is attractive as the two charges are of the same sign. Therefore, there will be a net attractive force and so the two objects should move toward each other in the new reference frame.

So in one reference frame there is no net force so the particles will not have net attraction. In another reference frame there is a net attractive force so they will attract each other.

Is there anything I'm missing here?

Apologies if this question is too elementary for this site, I'm just a high school physics student but I got into USAPhO.

asked Jun 10, 2021 in General Physics by KingLogic (0 points) [ revision history ]

1 Answer

+ 1 like - 0 dislike

No such attraction happens. Let's consider two point charges of charge $q$ at rest, and say $F_{grav} = F_{EM}$. 

If they are separated by a distance $d$ on the $x$-axis, the force felt by one charge from the other is 

$ \vec{F}_0 = q \vec{E}_0 = \frac{kq^2}{d^2}\hat{x}$

Let's transform to a frame traveling with velocity $\vec{v} = -v_0\hat{y}$. Now the particles appear to be traveling upwards with velocity $v_0$. As you say, this induces a magnetic field---along the line separating the particles it is:

$\vec{B} = \frac{ \vec{v}}{c} \times \vec{E} = \gamma\cdot \frac{ \vec{v}}{c}\times\vec{E_0}$

where we've used the transformed $\vec{E}$-field in the new frame: $\vec{E} = \gamma\vec{E}_0$. We can calculate the Lorentz force of one particle on the other:

$\vec{F} = q\vec{E} + q\frac{\vec{v}}{c}\times\vec{B} = q\vec{E}(1-v^2/c^2) = \frac{q\vec{E}_0}{\gamma}$

since the direction of travel is perpendicular to the electric field on the diameter of the charges. This is just equal to the electric force in the rest frame scaled by a factor $1/\gamma$ due to normal frame transformations. The same such transformation will happen to the gravitational force: perpendicular to the direction of motion, it will decrease by a factor of $1/\gamma$. The forces will be equal in both frames leading to no contradiction. 

If you didn't understand what I did above, then you may want to try another forum. But I can offer some less technical explanation. In moving frames, the electric field will appear to increase. This unbalances the moving charges, and would make them separate, if it were not for the attractive effect of the magnetic field. This brings them back into balance with the gravitational force. So in a slightly ironic fashion, it's the magnetic field (which you fear violates relativity) which is actually the crucial ingredient for preserving the predictions of relativity.

You can see this pdf by Kirk McDonald for a more complete derivation, and a list of classic references on this problem. 

answered Jun 11, 2021 by gorg (10 points) [ no revision ]

Your answer

Please use answers only to (at least partly) answer questions. To comment, discuss, or ask for clarification, leave a comment instead.
To mask links under text, please type your text, highlight it, and click the "link" button. You can then enter your link URL.
Please consult the FAQ for as to how to format your post.
This is the answer box; if you want to write a comment instead, please use the 'add comment' button.
Live preview (may slow down editor)   Preview
Your name to display (optional):
Privacy: Your email address will only be used for sending these notifications.
Anti-spam verification:
If you are a human please identify the position of the character covered by the symbol $\varnothing$ in the following word:
p$\hbar$ysicsOv$\varnothing$rflow
Then drag the red bullet below over the corresponding character of our banner. When you drop it there, the bullet changes to green (on slow internet connections after a few seconds).
Please complete the anti-spam verification




user contributions licensed under cc by-sa 3.0 with attribution required

Your rights
...